Difference between revisions of "2024 AMC 10A Problems/Problem 2"

Line 2: Line 2:
  
 
<math>\textbf{(A) } \text{R E D} \qquad\textbf{(B) } \text{L S T E R} \qquad\textbf{(C) } \text{Q E D} \qquad\textbf{(D) } \text{L M N O P} \qquad\textbf{(E) } 42 </math>
 
<math>\textbf{(A) } \text{R E D} \qquad\textbf{(B) } \text{L S T E R} \qquad\textbf{(C) } \text{Q E D} \qquad\textbf{(D) } \text{L M N O P} \qquad\textbf{(E) } 42 </math>
 +
 +
==Video Solution==
 +
https://m.youtube.com/watch?v=c4KTboqV_-o&pp=ygUJc3BlbGwgcmVk

Revision as of 01:41, 3 August 2024